Difference between revisions of "2023 AIME I Problems/Problem 15"

(Solution)
m
 
(19 intermediate revisions by 10 users not shown)
Line 1: Line 1:
==Problem 15==
+
==Problem==
 
Find the largest prime number <math>p<1000</math> for which there exists a complex number <math>z</math> satisfying
 
Find the largest prime number <math>p<1000</math> for which there exists a complex number <math>z</math> satisfying
* the real and imaginary parts of <math>z</math> are integers;
 
* <math>|z|=\sqrt{p}</math>, and
 
* there exists a triangle with side lengths <math>p</math>, the real part of <math>z^{3}</math>, and the imaginary part of <math>z^{3}</math>.
 
  
==Answer: 349==
+
* the real and imaginary part of <math>z</math> are both integers;
Suppose <math>z=a+bi</math>; notice that <math>\arg(z^{3})\approx 45^{\circ}</math>, so by De Moivre’s theorem <math>\arg(z)\approx 15^{\circ}</math> and <math>\tfrac{b}{a}\approx tan(15^{\circ})=2-\sqrt{3}</math>. Now just try pairs <math>(a, b)=(t, \left(2-\sqrt{3}\right)t)</math> going down from <math>t=\lfloor\sqrt{1000}\rfloor</math>, writing down the value of <math>a^{2}+b^{2}</math> on the right; and eventually we arrive at <math>(a, b)=(18, 5)</math> the first time <math>a^{2}+b^{2}</math> is prime. Therefore, <math>p=\boxed{349}</math>.
+
 
 +
* <math>|z|=\sqrt{p},</math> and
 +
 
 +
* there exists a triangle whose three side lengths are <math>p,</math> the real part of <math>z^{3},</math> and the imaginary part of <math>z^{3}.</math>
  
 
==Solution==
 
==Solution==
 +
Assume that <math>z=a+bi</math>. Then,
 +
<cmath>z^3=(a^3-3ab^2)+(3a^2b-b^3)i</cmath>Note that by the Triangle Inequality,
 +
<cmath>|(a^3-3ab^2)-(3a^2b-b^3)|<p\implies |a^3+b^3-3ab^2-3a^2b|<a^2+b^2</cmath>Thus, we know
 +
<cmath>|a+b||a^2+b^2-4ab|<a^2+b^2</cmath>Without loss of generality, assume <math>a>b</math> (as otherwise, consider <math>i^3\overline z=b+ai</math>). If <math>|a/b|\geq 4</math>, then
 +
<cmath>17b^2\geq a^2+b^2>|a+b||a^2+b^2-4ab|\geq |b-4b||16b^2-16b^2+b^2|=3b^3</cmath>`Thus, this means <math>b\leq\frac{17}3</math> or <math>b\leq 5</math>. Also note that the roots of <math>x^2-4x+1</math> are <math>2\pm\sqrt 3</math>, so thus if <math>b\geq 6</math>,
 +
<cmath>2\sqrt 3b=(2(2-\sqrt 3)-4)b<a<4b</cmath>Note that
 +
<cmath>1000>p=a^2+b^2\geq 12b^2+b^2=13b^2</cmath>so <math>b^2<81</math>, and <math>b<9</math>. If <math>b=8</math>, then <math>16\sqrt 3\leq a\leq 32</math>. Note that <math>\gcd(a,b)=1</math>, and <math>a\not\equiv b\pmod 2</math>, so <math>a=29</math> or <math>31</math>. However, then <math>5\mid a^2+b^2</math>, absurd.
 +
 +
If <math>b=7</math>, by similar logic, we have that <math>14\sqrt 3 <a< 28</math>, so <math>b=26</math>. However, once again, <math>5\mid a^2+b^2</math>. If <math>b=6</math>, by the same logic, <math>12\sqrt3<a<24</math>, so <math>a=23</math>, where we run into the same problem. Thus <math>b\leq 5</math> indeed.
 +
 +
If <math>b=5</math>, note that
 +
<cmath>(a+5)(a^2+25-20a)<a^2+25\implies a<20</cmath>We note that <math>p=5^2+18^2=349</math> works. Thus, we just need to make sure that if <math>b\leq 4</math>, <math>a\leq 18</math>. But this is easy, as
 +
<cmath>p>(a+b)(a^2+b^2-4ab)\geq (4+18)(4^2+18^2-4\cdot 4\cdot 18)>1000</cmath>absurd. Thus, the answer is <math>\boxed{349}</math>.
  
 +
 +
==Solution 2==
 
Denote <math>z = a + i b</math>. Thus, <math>a^2 + b^2 = p</math>.
 
Denote <math>z = a + i b</math>. Thus, <math>a^2 + b^2 = p</math>.
  
Line 19: Line 34:
 
Because <math>p</math>, <math>{\rm Re} \left( z^3 \right)</math>, <math>{\rm Im} \left( z^3 \right)</math> are three sides of a triangle, we have <math>{\rm Re} \left( z^3 \right) > 0</math> and <math>{\rm Im} \left( z^3 \right) > 0</math>.
 
Because <math>p</math>, <math>{\rm Re} \left( z^3 \right)</math>, <math>{\rm Im} \left( z^3 \right)</math> are three sides of a triangle, we have <math>{\rm Re} \left( z^3 \right) > 0</math> and <math>{\rm Im} \left( z^3 \right) > 0</math>.
 
Thus,
 
Thus,
\begin{eqnarray*}
+
<cmath>
 +
\begin{align*}
 
a \left( a^2 - 3 b^2 \right) & > 0 , \hspace{1cm} (1) \\
 
a \left( a^2 - 3 b^2 \right) & > 0 , \hspace{1cm} (1) \\
 
b \left( - b^2 + 3 a^2 \right) & > 0. \hspace{1cm} (2)
 
b \left( - b^2 + 3 a^2 \right) & > 0. \hspace{1cm} (2)
\end{eqnarray*}
+
\end{align*}
 +
</cmath>
  
 
Because <math>p</math>, <math>{\rm Re} \left( z^3 \right)</math>, <math>{\rm Im} \left( z^3 \right)</math> are three sides of a triangle, we have the following triangle inequalities:
 
Because <math>p</math>, <math>{\rm Re} \left( z^3 \right)</math>, <math>{\rm Im} \left( z^3 \right)</math> are three sides of a triangle, we have the following triangle inequalities:
 +
<cmath>
 
\begin{align*}
 
\begin{align*}
 
{\rm Re} \left( z^3 \right) + {\rm Im} \left( z^3 \right) & > p \hspace{1cm} (3) \\
 
{\rm Re} \left( z^3 \right) + {\rm Im} \left( z^3 \right) & > p \hspace{1cm} (3) \\
Line 30: Line 48:
 
p + {\rm Im} \left( z^3 \right) & > {\rm Re} \left( z^3 \right) \hspace{1cm} (5)
 
p + {\rm Im} \left( z^3 \right) & > {\rm Re} \left( z^3 \right) \hspace{1cm} (5)
 
\end{align*}
 
\end{align*}
 +
</cmath>
  
 
We notice that <math>| z^3 | = p^{3/2}</math>, and <math>{\rm Re} \left( z^3 \right)</math>, <math>{\rm Im} \left( z^3 \right)</math>, and <math>| z^3 |</math> form a right triangle. Thus, <math>{\rm Re} z^3 + {\rm Im} z^3 > p^{3/2}</math>.
 
We notice that <math>| z^3 | = p^{3/2}</math>, and <math>{\rm Re} \left( z^3 \right)</math>, <math>{\rm Im} \left( z^3 \right)</math>, and <math>| z^3 |</math> form a right triangle. Thus, <math>{\rm Re} z^3 + {\rm Im} z^3 > p^{3/2}</math>.
Line 36: Line 55:
  
 
Conditions (4) and (5) can be written in the joint form as
 
Conditions (4) and (5) can be written in the joint form as
\[
+
<cmath>
 
\left| {\rm Re} \left( z^3 \right) - {\rm Im} \left( z^3 \right) \right| < p . \hspace{1cm} (4)
 
\left| {\rm Re} \left( z^3 \right) - {\rm Im} \left( z^3 \right) \right| < p . \hspace{1cm} (4)
\]
+
</cmath>
 +
 
  
 
We have
 
We have
 +
<cmath>
 
\begin{align*}
 
\begin{align*}
 
{\rm Re} \left( z^3 \right) - {\rm Im} \left( z^3 \right)
 
{\rm Re} \left( z^3 \right) - {\rm Im} \left( z^3 \right)
Line 46: Line 67:
 
& = \left( a + b \right) \left( a^2 - 4 ab + b^2 \right)
 
& = \left( a + b \right) \left( a^2 - 4 ab + b^2 \right)
 
\end{align*}
 
\end{align*}
 +
</cmath>
 
and <math>p = a^2 + b^2</math>.
 
and <math>p = a^2 + b^2</math>.
  
 
Thus, (5) can be written as
 
Thus, (5) can be written as
\[
+
<cmath>
 
\left| \left( a + b \right) \left( a^2 - 4 ab + b^2 \right)  \right|
 
\left| \left( a + b \right) \left( a^2 - 4 ab + b^2 \right)  \right|
 
< a^2 + b^2 . \hspace{1cm} (6)
 
< a^2 + b^2 . \hspace{1cm} (6)
\]
+
</cmath>
  
 
Therefore, we need to jointly solve (1), (2), (6).
 
Therefore, we need to jointly solve (1), (2), (6).
Line 59: Line 81:
  
 
Thus, (1) and (2) are reduced to
 
Thus, (1) and (2) are reduced to
\[
+
<cmath>
 
a > \sqrt{3} b . \hspace{1cm} (7)
 
a > \sqrt{3} b . \hspace{1cm} (7)
\]
+
</cmath>
  
 
Let <math>a = \lambda b</math>. Plugging this into (6), we get
 
Let <math>a = \lambda b</math>. Plugging this into (6), we get
 +
<cmath>
 
\begin{align*}
 
\begin{align*}
 
\left| \left( \left( \lambda - 2 \right)^2 - 3 \right)  \right|
 
\left| \left( \left( \lambda - 2 \right)^2 - 3 \right)  \right|
 
< \frac{1}{b} \frac{\lambda^2 + 1}{\lambda + 1} .  \hspace{1cm} (8)
 
< \frac{1}{b} \frac{\lambda^2 + 1}{\lambda + 1} .  \hspace{1cm} (8)
 
\end{align*}
 
\end{align*}
 +
</cmath>
  
 
Because <math>p= a^2 + b^2</math> is a prime, <math>a</math> and <math>b</math> are relatively prime.
 
Because <math>p= a^2 + b^2</math> is a prime, <math>a</math> and <math>b</math> are relatively prime.
Line 74: Line 98:
  
 
To facilitate efficient search, we apply the following criteria:
 
To facilitate efficient search, we apply the following criteria:
\begin{enumerate}
+
 
 +
\begin{enumerate*}
 
\item To satisfy (7) and <math>a^2 + b^2 < 1000</math>, we have <math>1 \leq b \leq 15</math>.
 
\item To satisfy (7) and <math>a^2 + b^2 < 1000</math>, we have <math>1 \leq b \leq 15</math>.
 
In the outer layer, we search for <math>b</math> in a decreasing order.
 
In the outer layer, we search for <math>b</math> in a decreasing order.
Line 85: Line 110:
 
Thus, <math>\sqrt{3} < \lambda \leq \frac{30}{9}</math>.
 
Thus, <math>\sqrt{3} < \lambda \leq \frac{30}{9}</math>.
 
Thus, the R.H.S. of (8) has the following upper bound
 
Thus, the R.H.S. of (8) has the following upper bound
 +
<cmath>
 
\begin{align*}
 
\begin{align*}
 
\frac{1}{b} \frac{\lambda^2 + 1}{\lambda + 1}
 
\frac{1}{b} \frac{\lambda^2 + 1}{\lambda + 1}
Line 92: Line 118:
 
& < \frac{10}{27} .
 
& < \frac{10}{27} .
 
\end{align*}
 
\end{align*}
 +
</cmath>
  
 
Hence, to satisfy (8), a necessary condition is
 
Hence, to satisfy (8), a necessary condition is
 +
<cmath>
 
\begin{align*}
 
\begin{align*}
 
\left| \left( \left( \lambda - 2 \right)^2 - 3 \right)  \right|
 
\left| \left( \left( \lambda - 2 \right)^2 - 3 \right)  \right|
 
< \frac{10}{27} .
 
< \frac{10}{27} .
 
\end{align*}
 
\end{align*}
 +
</cmath>
  
 
However, this cannot be satisfied for <math>\sqrt{3} < \lambda \leq \frac{30}{9}</math>.
 
However, this cannot be satisfied for <math>\sqrt{3} < \lambda \leq \frac{30}{9}</math>.
Line 106: Line 135:
  
 
\item We use the following criteria to quickly eliminate <math>a</math> that make <math>a^2 + b^2</math> a composite number.
 
\item We use the following criteria to quickly eliminate <math>a</math> that make <math>a^2 + b^2</math> a composite number.
\begin{enumerate}
+
 
\item For <math>b \equiv 1 \pmod{2}</math>, we eliminate <math>a</math> satisfying <math>a \equiv 1 \pmod{2}</math>.
+
* For <math>b \equiv 1 \pmod{2}</math>, we eliminate <math>a</math> satisfying <math>a \equiv 1 \pmod{2}</math>.
\item For <math>b \equiv \pm 1 \pmod{5}</math> (resp. <math>b \equiv \pm 2 \pmod{5}</math>), we eliminate <math>a</math> satisfying <math>a \equiv \pm 2 \pmod{5}</math> (resp. <math>a \equiv \pm 1 \pmod{5}</math>).
+
 
\end{enumerate}
+
* For <math>b \equiv \pm 1 \pmod{5}</math> (resp. <math>b \equiv \pm 2 \pmod{5}</math>), we eliminate <math>a</math> satisfying <math>a \equiv \pm 2 \pmod{5}</math> (resp. <math>a \equiv \pm 1 \pmod{5}</math>).
 +
 
  
 
\item For the remaining <math>\left( b, a \right)</math>, check whether (8) and the condition that <math>a^2 + b^2</math> is prime are both satisfied.
 
\item For the remaining <math>\left( b, a \right)</math>, check whether (8) and the condition that <math>a^2 + b^2</math> is prime are both satisfied.
Line 117: Line 147:
  
 
\item For the remaining search, given <math>b</math>, we only search for <math>a \geq \sqrt{349 - b^2}</math>.
 
\item For the remaining search, given <math>b</math>, we only search for <math>a \geq \sqrt{349 - b^2}</math>.
\end{enumerate}
 
  
 
Following the above search criteria, we find the final answer as <math>b = 5</math> and <math>a = 18</math>.
 
Following the above search criteria, we find the final answer as <math>b = 5</math> and <math>a = 18</math>.
Line 123: Line 152:
  
 
~Steven Chen (Professor Chen Education Palace, www.professorchenedu.com)
 
~Steven Chen (Professor Chen Education Palace, www.professorchenedu.com)
 +
 +
==Video Solution==
 +
 +
https://youtu.be/V0KFMIXmp08
 +
 +
~Steven Chen (Professor Chen Education Palace, www.professorchenedu.com)
 +
 +
 +
==Video Solution==
 +
 +
https://youtu.be/tELK8fy36bs
 +
 +
~MathProblemSolvingSkills.com
 +
 +
 +
==Animated Video Solution==
 +
 +
https://youtu.be/1Y8ql7eHt34
 +
 +
~Star League (https://starleague.us)
 +
 +
 +
==See also==
 +
{{AIME box|year=2023|n=I|num-b=14|after=Last Problem}}
 +
 +
[[Category:Intermediate Algebra Problems]]
 +
[[Category:Intermediate Number Theory Problems]]
 +
{{MAA Notice}}

Latest revision as of 17:37, 3 June 2023

Problem

Find the largest prime number $p<1000$ for which there exists a complex number $z$ satisfying

  • the real and imaginary part of $z$ are both integers;
  • $|z|=\sqrt{p},$ and
  • there exists a triangle whose three side lengths are $p,$ the real part of $z^{3},$ and the imaginary part of $z^{3}.$

Solution

Assume that $z=a+bi$. Then, \[z^3=(a^3-3ab^2)+(3a^2b-b^3)i\]Note that by the Triangle Inequality, \[|(a^3-3ab^2)-(3a^2b-b^3)|<p\implies |a^3+b^3-3ab^2-3a^2b|<a^2+b^2\]Thus, we know \[|a+b||a^2+b^2-4ab|<a^2+b^2\]Without loss of generality, assume $a>b$ (as otherwise, consider $i^3\overline z=b+ai$). If $|a/b|\geq 4$, then \[17b^2\geq a^2+b^2>|a+b||a^2+b^2-4ab|\geq |b-4b||16b^2-16b^2+b^2|=3b^3\]`Thus, this means $b\leq\frac{17}3$ or $b\leq 5$. Also note that the roots of $x^2-4x+1$ are $2\pm\sqrt 3$, so thus if $b\geq 6$, \[2\sqrt 3b=(2(2-\sqrt 3)-4)b<a<4b\]Note that \[1000>p=a^2+b^2\geq 12b^2+b^2=13b^2\]so $b^2<81$, and $b<9$. If $b=8$, then $16\sqrt 3\leq a\leq 32$. Note that $\gcd(a,b)=1$, and $a\not\equiv b\pmod 2$, so $a=29$ or $31$. However, then $5\mid a^2+b^2$, absurd.

If $b=7$, by similar logic, we have that $14\sqrt 3 <a< 28$, so $b=26$. However, once again, $5\mid a^2+b^2$. If $b=6$, by the same logic, $12\sqrt3<a<24$, so $a=23$, where we run into the same problem. Thus $b\leq 5$ indeed.

If $b=5$, note that \[(a+5)(a^2+25-20a)<a^2+25\implies a<20\]We note that $p=5^2+18^2=349$ works. Thus, we just need to make sure that if $b\leq 4$, $a\leq 18$. But this is easy, as \[p>(a+b)(a^2+b^2-4ab)\geq (4+18)(4^2+18^2-4\cdot 4\cdot 18)>1000\]absurd. Thus, the answer is $\boxed{349}$.


Solution 2

Denote $z = a + i b$. Thus, $a^2 + b^2 = p$.

Thus, \[z^3 = a \left( a^2 - 3 b^2 \right) + i b \left( - b^2 + 3 a^2 \right) .\]

Because $p$, ${\rm Re} \left( z^3 \right)$, ${\rm Im} \left( z^3 \right)$ are three sides of a triangle, we have ${\rm Re} \left( z^3 \right) > 0$ and ${\rm Im} \left( z^3 \right) > 0$. Thus, \begin{align*} a \left( a^2 - 3 b^2 \right) & > 0 , \hspace{1cm} (1) \\ b \left( - b^2 + 3 a^2 \right) & > 0. \hspace{1cm} (2) \end{align*}

Because $p$, ${\rm Re} \left( z^3 \right)$, ${\rm Im} \left( z^3 \right)$ are three sides of a triangle, we have the following triangle inequalities: \begin{align*} {\rm Re} \left( z^3 \right) + {\rm Im} \left( z^3 \right) & > p \hspace{1cm} (3) \\ p + {\rm Re} \left( z^3 \right) & > {\rm Im} \left( z^3 \right) \hspace{1cm} (4) \\ p + {\rm Im} \left( z^3 \right) & > {\rm Re} \left( z^3 \right) \hspace{1cm} (5) \end{align*}

We notice that $| z^3 | = p^{3/2}$, and ${\rm Re} \left( z^3 \right)$, ${\rm Im} \left( z^3 \right)$, and $| z^3 |$ form a right triangle. Thus, ${\rm Re} z^3 + {\rm Im} z^3 > p^{3/2}$. Because $p > 1$, $p^{3/2} > p$. Therefore, (3) holds.

Conditions (4) and (5) can be written in the joint form as \[\left| {\rm Re} \left( z^3 \right) - {\rm Im} \left( z^3 \right) \right| < p . \hspace{1cm} (4)\]


We have \begin{align*} {\rm Re} \left( z^3 \right) - {\rm Im} \left( z^3 \right) & = \left( a^3 - 3 a b^2 \right) - \left( - b^3 + 3 a^2 b \right) \\ & = \left( a + b \right) \left( a^2 - 4 ab + b^2 \right) \end{align*} and $p = a^2 + b^2$.

Thus, (5) can be written as \[\left| \left( a + b \right) \left( a^2 - 4 ab + b^2 \right)  \right| < a^2 + b^2 . \hspace{1cm} (6)\]

Therefore, we need to jointly solve (1), (2), (6). From (1) and (2), we have either $a, b >0$, or $a, b < 0$. In (6), by symmetry, without loss of generality, we assume $a, b > 0$.

Thus, (1) and (2) are reduced to \[a > \sqrt{3} b . \hspace{1cm} (7)\]

Let $a = \lambda b$. Plugging this into (6), we get \begin{align*} \left| \left( \left( \lambda - 2 \right)^2 - 3 \right)  \right| < \frac{1}{b} \frac{\lambda^2 + 1}{\lambda + 1} .  \hspace{1cm} (8) \end{align*}

Because $p= a^2 + b^2$ is a prime, $a$ and $b$ are relatively prime.

Therefore, we can use (7), (8), $a^2 + b^2 <1000$, and $a$ and $b$ are relatively prime to solve the problem.

To facilitate efficient search, we apply the following criteria:

\begin{enumerate*} \item To satisfy (7) and $a^2 + b^2 < 1000$, we have $1 \leq b \leq 15$. In the outer layer, we search for $b$ in a decreasing order. In the inner layer, for each given $b$, we search for $a$. \item Given $b$, we search for $a$ in the range $\sqrt{3} b < a < \sqrt{1000 - b^2}$. \item We can prove that for $b \geq 9$, there is no feasible $a$. The proof is as follows.

For $b \geq 9$, to satisfy $a^2 + b^2 < 1000$, we have $a \leq 30$. Thus, $\sqrt{3} < \lambda \leq \frac{30}{9}$. Thus, the R.H.S. of (8) has the following upper bound \begin{align*} \frac{1}{b} \frac{\lambda^2 + 1}{\lambda + 1} & < \frac{1}{b} \frac{\lambda^2 + \lambda}{\lambda + 1} \\ & = \frac{\lambda}{b} \\ & \leq \frac{\frac{30}{9}}{9} \\ & < \frac{10}{27} . \end{align*}

Hence, to satisfy (8), a necessary condition is \begin{align*} \left| \left( \left( \lambda - 2 \right)^2 - 3 \right)  \right| < \frac{10}{27} . \end{align*}

However, this cannot be satisfied for $\sqrt{3} < \lambda \leq \frac{30}{9}$. Therefore, there is no feasible solution for $b \geq 9$. Therefore, we only need to consider $b \leq 8$.

\item We eliminate $a$ that are not relatively prime to $b$.

\item We use the following criteria to quickly eliminate $a$ that make $a^2 + b^2$ a composite number.

  • For $b \equiv 1 \pmod{2}$, we eliminate $a$ satisfying $a \equiv 1 \pmod{2}$.
  • For $b \equiv \pm 1 \pmod{5}$ (resp. $b \equiv \pm 2 \pmod{5}$), we eliminate $a$ satisfying $a \equiv \pm 2 \pmod{5}$ (resp. $a \equiv \pm 1 \pmod{5}$).


\item For the remaining $\left( b, a \right)$, check whether (8) and the condition that $a^2 + b^2$ is prime are both satisfied.

The first feasible solution is $b = 5$ and $a = 18$. Thus, $a^2 + b^2 = 349$.

\item For the remaining search, given $b$, we only search for $a \geq \sqrt{349 - b^2}$.

Following the above search criteria, we find the final answer as $b = 5$ and $a = 18$. Thus, the largest prime $p$ is $p = a^2 + b^2 = \boxed{\textbf{(349) }}$.

~Steven Chen (Professor Chen Education Palace, www.professorchenedu.com)

Video Solution

https://youtu.be/V0KFMIXmp08

~Steven Chen (Professor Chen Education Palace, www.professorchenedu.com)


Video Solution

https://youtu.be/tELK8fy36bs

~MathProblemSolvingSkills.com


Animated Video Solution

https://youtu.be/1Y8ql7eHt34

~Star League (https://starleague.us)


See also

2023 AIME I (ProblemsAnswer KeyResources)
Preceded by
Problem 14
Followed by
Last Problem
1 2 3 4 5 6 7 8 9 10 11 12 13 14 15
All AIME Problems and Solutions

The problems on this page are copyrighted by the Mathematical Association of America's American Mathematics Competitions. AMC logo.png